LSAT and Law School Admissions Forum

Get expert LSAT preparation and law school admissions advice from PowerScore Test Preparation.

User avatar
 Dave Killoran
PowerScore Staff
  • PowerScore Staff
  • Posts: 5852
  • Joined: Mar 25, 2011
|
#88169
Complete Question Explanation
(The complete setup for this game can be found here: lsat/viewtopic.php?f=170&p=88166#p88166)

The correct answer choice is (D).

A brief glance at the answer choices reveals that the form of the answers is somewhat unique: each answer specifies that the CD in question is the “only” CD to receive the specified rating. Although we will examine the distribution in a moment, the best starting point in a question of this type is to refer to the work done in previous questions:

Answer choice (A) can be disproved by the work done in question #9, which revealed that Q could be the only CD to receive three stars.

Answer choice (B) can be disproved by the work done in questions #8 and #9, which revealed that Q could be the only CD to receive four stars.

Answer choice (C) can be disproved by the question stem in question #11, which shows the value of waiting until the end of the game to attack Global questions.

Thus, by referring to questions #8 and #9, we can eliminate answer choices (A) and (B). If you did not wait until the end of the game to come back to this question, and answer choice (C) is still in contention for you, do not despair. Answer choices (C), (D), and (E) each involve R being the only CD to receive a particular rating. Thus, let’s examine what occurs when R is the only CD to receive a certain rating:

When R is the only CD to receive a particular rating, then from the third rule we can infer that H and I must receive the same rating, with H receiving one more star than N:
G2-Q10-d1.png

Although Q and S have yet to be considered, the information above is actually sufficient to solve this question. Because I can never receive four stars (as discussed in the setup), this block has only two placement options: 1-2 and 2-3. Accordingly, either N or the HI block always receive two stars. As R must be the only CD to receive its rating, R can therefore never receive two stars, and answer choice (D) is correct.
You do not have the required permissions to view the files attached to this post.
 lazidaisies
  • Posts: 1
  • Joined: May 10, 2020
|
#75418
Hello,

My first instinct was to narrow choices down to those with Q, which I believed to be the most restricted variable. If I was running low on time I would've gone with B, but I would have been wrong. I was able to get to the right answer by testing the rest of the choices (making hypotheticals for each one, since none of my previous set-ups had R alone), but I was wondering if there was a more strategic way of attacking this one that wasn't so time-consuming?

Thanks so much!
 Paul Marsh
PowerScore Staff
  • PowerScore Staff
  • Posts: 290
  • Joined: Oct 15, 2019
|
#75526
Hi lazidaisies! Great question - whenever you find yourself testing out all the answer choices, you should always ask yourself, "How could I have done this faster?".

This is a Cannot be True question, so we're looking for an answer choice that would violate our rules. You're right that an answer choice putting Q in the wrong spot would work, since our last rule makes it clear that Q has to go third or fourth. So if you see an answer choice with Q first or second, of course we'd pick that right away. However, that would be a bit too easy for the LSAT. It's pretty rare that an answer to a Cannot be True question would be a clear-cut violation of a single rule like that, because that's basically a free point as far as the LSAC is concerned! So as I'm preparing to attack the answer choices, I probably wouldn't expect the answer to involve Q. As a result, I wouldn't look at Answer Choices (A) and (B) too closely after I notice that they put Q in its proper spot (3rd or 4th).

Instead, I would hone in on the block of [NH] that our second rule creates. On Linear Games, a block like that is an absolute magnet for creating scenarios that violate the rules in less obvious ways. So before I even peek at the answer choices, I'm already on the lookout any scenario that would mess with the [NH] block. Variable R going in the middle of the game seems like it could potentially screw with that block, so after briefly reading all the answer choices I'd seriously look at (D) and (E). (C) seems less likely to force the [NH] block apart, plus if you've already done Question 11 (which I'd recommend - doing Local questions first will give you new diagrams that can help with global questions) then you know there's no problem with R being the only CD to have one star. So I'd test out (D) and (E), in either order, to see if that forces N and H to be apart. Sure enough, (D) does.

The take-away here is that blocks in Linear Games are extremely limiting, and something to keep your eye on very closely.

Hope that helps!

Get the most out of your LSAT Prep Plus subscription.

Analyze and track your performance with our Testing and Analytics Package.